English Deutsch Français Italiano Español Português 繁體中文 Bahasa Indonesia Tiếng Việt ภาษาไทย
All categories

1 answers

Seems difficult to believe. Do you mean bounded away from 0? If this is the case 1/f will be bounded above (and if you think about these sorts of things, measurable).

2007-02-22 12:09:10 · answer #1 · answered by Sean H 5 · 0 0

fedest.com, questions and answers